How Do You Apply the Chain Rule in This Multivariable Calculus Problem?

Click For Summary
SUMMARY

The discussion focuses on applying the chain rule in multivariable calculus, specifically in the context of the function defined as ##f(x,y,z)=x^nf(1,y/x,z/x)##. Participants clarify that to prove ##f(tx,ty,tz)=t^nf(x,y,z)##, one must correctly substitute variables into the initial equation. The key takeaway is that by applying the chain rule correctly, users can derive the necessary relationships to complete the proof.

PREREQUISITES
  • Understanding of multivariable calculus concepts
  • Familiarity with the chain rule in calculus
  • Ability to manipulate and substitute variables in mathematical functions
  • Knowledge of function notation and properties
NEXT STEPS
  • Study the application of the chain rule in multivariable functions
  • Explore proofs involving homogeneous functions
  • Learn about substitution techniques in calculus
  • Review examples of function transformations in multivariable calculus
USEFUL FOR

Students and educators in mathematics, particularly those studying calculus, as well as anyone looking to deepen their understanding of multivariable function proofs and the chain rule application.

dakota1234
Messages
3
Reaction score
1
Moved from a technical forum, so homework template missing
I've been working on this one for a little bit, and I know I really just need to use the chain rule to solve it, but I can't seem to figure out how to set it up properly. Probably a dumb question, but I could really use some help on this!
Screen Shot 2018-11-26 at 4.57.55 PM.png
 

Attachments

  • Screen Shot 2018-11-26 at 4.57.55 PM.png
    Screen Shot 2018-11-26 at 4.57.55 PM.png
    10.4 KB · Views: 643
Physics news on Phys.org
I was able to work out a more generalized version of the proof, but I'm not sure how to apply it
Screen Shot 2018-11-26 at 5.03.55 PM.png
 

Attachments

  • Screen Shot 2018-11-26 at 5.03.55 PM.png
    Screen Shot 2018-11-26 at 5.03.55 PM.png
    84.7 KB · Views: 447
  • Like
Likes Delta2
you are given that ##f(x,y,z)=x^nf(1,y/x,z/x)## (1). If from (1) you can prove that ##f(tx,ty,tz)=t^nf(x,y,z)## (2) then together with the work already done by you , you ll be finished.

Now if you apply (1) for ##x=tx,y=ty,z=tz## what do you get? if you apply it correctly you ll get a result that if you apply (1) again (as it is directly, without substitution) you ll be able to prove (2).
 
Last edited:
Delta2 said:
you are given that ##f(x,y,z)=x^nf(1,y/x,z/x)## (1). If from (1) you can prove that ##f(tx,ty,tz)=t^nf(x,y,z)## (2) then together with the work already done by you , you ll be finished.

Now if you apply (1) for ##x=tx,y=ty,z=tz## what do you get? if you apply it correctly you ll get a result that if you apply (1) again (as it is directly, without substitution) you ll be able to prove (2).
I may just be really tired, but can you explain in more detail how "to apply it correctly"? I've tried plugging in the substitutions for x,y,z, since before I even posted on this forum, and I don't understand what I'm doing wrong.
 
Ok, let's say that ##f(u,v,w)=u^nf(1,v/u,w/u)## (1)

Applying (1) for ##u=tx,v=ty,w=tz## we get that ##f(tx,ty,tz)=(tx)^nf(1,\frac{ty}{tx},\frac{tz}{tx})=t^nx^nf(1,y/x,z/x)##

Now i have helped too much i think you must be able to see the last step, what is ##x^nf(1,y/x,z/x)## equal to?
 
Question: A clock's minute hand has length 4 and its hour hand has length 3. What is the distance between the tips at the moment when it is increasing most rapidly?(Putnam Exam Question) Answer: Making assumption that both the hands moves at constant angular velocities, the answer is ## \sqrt{7} .## But don't you think this assumption is somewhat doubtful and wrong?

Similar threads

  • · Replies 4 ·
Replies
4
Views
1K
  • · Replies 3 ·
Replies
3
Views
2K
Replies
2
Views
2K
  • · Replies 4 ·
Replies
4
Views
2K
Replies
2
Views
3K
Replies
4
Views
2K
  • · Replies 17 ·
Replies
17
Views
9K
  • · Replies 9 ·
Replies
9
Views
5K
  • · Replies 1 ·
Replies
1
Views
3K
  • · Replies 3 ·
Replies
3
Views
2K